Harmonic functions on $S^2$












4












$begingroup$


Consider the sphere $S^2 = lbrace (x,y,z) : x^2 + y^2 + z^2 = 1 rbrace$. This is a smooth manifold in $mathbb{R}^3$, and for a given point $s in S^2$, one can consider its coordinate neighborhood.



There are many ways to put a smooth structure on $S^2$, but all require at least two coordinate neighborhoods. One simple way is to make use of the local coordinates $theta, phi$ (azimuthal angle and inclination angle). For $s in U = S^2 setminus {(0,0,1), (0,0,-1)}$, define a coordinate map by



$ g : (0,2pi) times (0,pi) rightarrow U $



with formula given by



$ g(theta, phi) = (sin(phi) cos(theta), sin(phi) sin(theta), cos(phi)) $



This map is one-to-one, onto, and bicontinuous, so it is a homeomorphism. One can construct a similar map onto $V = S^2 setminus {(1,0,0),(-1,0,0)}$, so the whole sphere is covered. The fact that this defines a differentiable structure is easy to work out.



The question I want to ask is, are there any non-constant solutions to the equation



$ frac{partial^2 f}{partial theta^2} + frac{partial^2 f}{partial phi^2} = 0 $



where $f : S^2 rightarrow R$ is a smooth function.



Furthermore, what sorts of boundary conditions are necessary in order to ensure uniqueness.










share|cite|improve this question











$endgroup$








  • 2




    $begingroup$
    But your expression $partial^2 f/partial theta^2 + partial^2 f/partial phi^2$ is not the Laplacian on a sphere! So your equation is not consistent with your title. (Also, you probably want to ask about whether there are non-constant solutions to the equation in question.)
    $endgroup$
    – Rahul
    Nov 1 '10 at 10:15










  • $begingroup$
    Where do you want the solution to live? Since $theta,phi$ is only defined in a coordinate chart, do you mean for the function $f$ to be a map from $U to mathbb{C}$? Do you require the function to extend to the north/south poles in any particular way? Continuously? With continuous derivatives?
    $endgroup$
    – Willie Wong
    Nov 1 '10 at 10:20












  • $begingroup$
    These are both good comments and I will edit my question.
    $endgroup$
    – Eric Haengel
    Nov 1 '10 at 10:46
















4












$begingroup$


Consider the sphere $S^2 = lbrace (x,y,z) : x^2 + y^2 + z^2 = 1 rbrace$. This is a smooth manifold in $mathbb{R}^3$, and for a given point $s in S^2$, one can consider its coordinate neighborhood.



There are many ways to put a smooth structure on $S^2$, but all require at least two coordinate neighborhoods. One simple way is to make use of the local coordinates $theta, phi$ (azimuthal angle and inclination angle). For $s in U = S^2 setminus {(0,0,1), (0,0,-1)}$, define a coordinate map by



$ g : (0,2pi) times (0,pi) rightarrow U $



with formula given by



$ g(theta, phi) = (sin(phi) cos(theta), sin(phi) sin(theta), cos(phi)) $



This map is one-to-one, onto, and bicontinuous, so it is a homeomorphism. One can construct a similar map onto $V = S^2 setminus {(1,0,0),(-1,0,0)}$, so the whole sphere is covered. The fact that this defines a differentiable structure is easy to work out.



The question I want to ask is, are there any non-constant solutions to the equation



$ frac{partial^2 f}{partial theta^2} + frac{partial^2 f}{partial phi^2} = 0 $



where $f : S^2 rightarrow R$ is a smooth function.



Furthermore, what sorts of boundary conditions are necessary in order to ensure uniqueness.










share|cite|improve this question











$endgroup$








  • 2




    $begingroup$
    But your expression $partial^2 f/partial theta^2 + partial^2 f/partial phi^2$ is not the Laplacian on a sphere! So your equation is not consistent with your title. (Also, you probably want to ask about whether there are non-constant solutions to the equation in question.)
    $endgroup$
    – Rahul
    Nov 1 '10 at 10:15










  • $begingroup$
    Where do you want the solution to live? Since $theta,phi$ is only defined in a coordinate chart, do you mean for the function $f$ to be a map from $U to mathbb{C}$? Do you require the function to extend to the north/south poles in any particular way? Continuously? With continuous derivatives?
    $endgroup$
    – Willie Wong
    Nov 1 '10 at 10:20












  • $begingroup$
    These are both good comments and I will edit my question.
    $endgroup$
    – Eric Haengel
    Nov 1 '10 at 10:46














4












4








4


1



$begingroup$


Consider the sphere $S^2 = lbrace (x,y,z) : x^2 + y^2 + z^2 = 1 rbrace$. This is a smooth manifold in $mathbb{R}^3$, and for a given point $s in S^2$, one can consider its coordinate neighborhood.



There are many ways to put a smooth structure on $S^2$, but all require at least two coordinate neighborhoods. One simple way is to make use of the local coordinates $theta, phi$ (azimuthal angle and inclination angle). For $s in U = S^2 setminus {(0,0,1), (0,0,-1)}$, define a coordinate map by



$ g : (0,2pi) times (0,pi) rightarrow U $



with formula given by



$ g(theta, phi) = (sin(phi) cos(theta), sin(phi) sin(theta), cos(phi)) $



This map is one-to-one, onto, and bicontinuous, so it is a homeomorphism. One can construct a similar map onto $V = S^2 setminus {(1,0,0),(-1,0,0)}$, so the whole sphere is covered. The fact that this defines a differentiable structure is easy to work out.



The question I want to ask is, are there any non-constant solutions to the equation



$ frac{partial^2 f}{partial theta^2} + frac{partial^2 f}{partial phi^2} = 0 $



where $f : S^2 rightarrow R$ is a smooth function.



Furthermore, what sorts of boundary conditions are necessary in order to ensure uniqueness.










share|cite|improve this question











$endgroup$




Consider the sphere $S^2 = lbrace (x,y,z) : x^2 + y^2 + z^2 = 1 rbrace$. This is a smooth manifold in $mathbb{R}^3$, and for a given point $s in S^2$, one can consider its coordinate neighborhood.



There are many ways to put a smooth structure on $S^2$, but all require at least two coordinate neighborhoods. One simple way is to make use of the local coordinates $theta, phi$ (azimuthal angle and inclination angle). For $s in U = S^2 setminus {(0,0,1), (0,0,-1)}$, define a coordinate map by



$ g : (0,2pi) times (0,pi) rightarrow U $



with formula given by



$ g(theta, phi) = (sin(phi) cos(theta), sin(phi) sin(theta), cos(phi)) $



This map is one-to-one, onto, and bicontinuous, so it is a homeomorphism. One can construct a similar map onto $V = S^2 setminus {(1,0,0),(-1,0,0)}$, so the whole sphere is covered. The fact that this defines a differentiable structure is easy to work out.



The question I want to ask is, are there any non-constant solutions to the equation



$ frac{partial^2 f}{partial theta^2} + frac{partial^2 f}{partial phi^2} = 0 $



where $f : S^2 rightarrow R$ is a smooth function.



Furthermore, what sorts of boundary conditions are necessary in order to ensure uniqueness.







real-analysis ordinary-differential-equations differential-geometry






share|cite|improve this question















share|cite|improve this question













share|cite|improve this question




share|cite|improve this question








edited Nov 1 '10 at 10:47







Eric Haengel

















asked Nov 1 '10 at 9:51









Eric HaengelEric Haengel

3,01411530




3,01411530








  • 2




    $begingroup$
    But your expression $partial^2 f/partial theta^2 + partial^2 f/partial phi^2$ is not the Laplacian on a sphere! So your equation is not consistent with your title. (Also, you probably want to ask about whether there are non-constant solutions to the equation in question.)
    $endgroup$
    – Rahul
    Nov 1 '10 at 10:15










  • $begingroup$
    Where do you want the solution to live? Since $theta,phi$ is only defined in a coordinate chart, do you mean for the function $f$ to be a map from $U to mathbb{C}$? Do you require the function to extend to the north/south poles in any particular way? Continuously? With continuous derivatives?
    $endgroup$
    – Willie Wong
    Nov 1 '10 at 10:20












  • $begingroup$
    These are both good comments and I will edit my question.
    $endgroup$
    – Eric Haengel
    Nov 1 '10 at 10:46














  • 2




    $begingroup$
    But your expression $partial^2 f/partial theta^2 + partial^2 f/partial phi^2$ is not the Laplacian on a sphere! So your equation is not consistent with your title. (Also, you probably want to ask about whether there are non-constant solutions to the equation in question.)
    $endgroup$
    – Rahul
    Nov 1 '10 at 10:15










  • $begingroup$
    Where do you want the solution to live? Since $theta,phi$ is only defined in a coordinate chart, do you mean for the function $f$ to be a map from $U to mathbb{C}$? Do you require the function to extend to the north/south poles in any particular way? Continuously? With continuous derivatives?
    $endgroup$
    – Willie Wong
    Nov 1 '10 at 10:20












  • $begingroup$
    These are both good comments and I will edit my question.
    $endgroup$
    – Eric Haengel
    Nov 1 '10 at 10:46








2




2




$begingroup$
But your expression $partial^2 f/partial theta^2 + partial^2 f/partial phi^2$ is not the Laplacian on a sphere! So your equation is not consistent with your title. (Also, you probably want to ask about whether there are non-constant solutions to the equation in question.)
$endgroup$
– Rahul
Nov 1 '10 at 10:15




$begingroup$
But your expression $partial^2 f/partial theta^2 + partial^2 f/partial phi^2$ is not the Laplacian on a sphere! So your equation is not consistent with your title. (Also, you probably want to ask about whether there are non-constant solutions to the equation in question.)
$endgroup$
– Rahul
Nov 1 '10 at 10:15












$begingroup$
Where do you want the solution to live? Since $theta,phi$ is only defined in a coordinate chart, do you mean for the function $f$ to be a map from $U to mathbb{C}$? Do you require the function to extend to the north/south poles in any particular way? Continuously? With continuous derivatives?
$endgroup$
– Willie Wong
Nov 1 '10 at 10:20






$begingroup$
Where do you want the solution to live? Since $theta,phi$ is only defined in a coordinate chart, do you mean for the function $f$ to be a map from $U to mathbb{C}$? Do you require the function to extend to the north/south poles in any particular way? Continuously? With continuous derivatives?
$endgroup$
– Willie Wong
Nov 1 '10 at 10:20














$begingroup$
These are both good comments and I will edit my question.
$endgroup$
– Eric Haengel
Nov 1 '10 at 10:46




$begingroup$
These are both good comments and I will edit my question.
$endgroup$
– Eric Haengel
Nov 1 '10 at 10:46










3 Answers
3






active

oldest

votes


















7












$begingroup$

Since $f$ is a smooth function from $S^2to mathbb{R}$, it is a smooth function on your coordinate chart. It is easy to see that by periodicity, you also have that $f$ is a smooth function from $S^1times(0,pi)tomathbb{R}$.



Since $f$ is smooth on $S^2$, the limits



$$ lim_{phi to 0, pi} frac{1}{sin phi} frac{partial}{partial theta} f $$



need to exist and be finite: this is the requirement that $f$ is smooth in "the other chart". Therefore you have that $partial_theta f to 0$ as $phi$ approach either boundary.



Your equation, however, requires that $partial_theta f$ be a harmonic function on $S^1times (0,pi)$, a compact domain with boundary. Using the maximum principle, you immediately have that $partial_theta f = 0$ everywhere. Plugging back into the equation, this implies $partial^2f/partial phi^2 = 0$.



And so $f$ must be a linear function in $phi$. And we now apply the smoothness constraint yet again:



At the north pole, smoothness will require that



$$ lim_{phi to 0} partial_phi f(theta,phi) = lim_{phi to 0} frac{1}{sin phi} partial_theta f(theta + pi/2, phi) = 0$$



which implies that the constant slope of the linear function $f(phi)$ is 0. And hence $f$ must be the constant function.






share|cite|improve this answer









$endgroup$





















    7












    $begingroup$

    If I understand your question correctly, you are aware that $frac{partial^2 f}{partial theta^2} + frac{partial^2 f}{partial phi^2}$ is not actually the Laplacian on a sphere, but you want to know whether there are any nontrivial smooth functions on $S^2$ which are harmonic with respect to this modified Laplacian. The answer is no.



    A function $f(theta,phi)$ which satisfies $frac{partial^2 f}{partial theta^2} + frac{partial^2 f}{partial phi^2} = 0$ is really a harmonic function on the rectangle $[0,2pi] times [0,pi]$, which is mapped onto the sphere via $g^{-1}$. Continuity on the sphere requires that $f(theta,0) = text{const}$ (continuity at the north pole), $f(theta,pi) = text{const}$ (continuity at the south pole), and $f(0,phi) = f(2pi,phi) = h(phi)$ for some $h:[0,pi]rightarrow mathbb{R}$ (continuity across the prime meridian). For any $h$, the boundary conditions are independent of $theta$, so $f$ is a function of $phi$ alone. But then $frac{partial^2 f}{partial phi^2} = 0$ implies that $f$ is linear in $phi$. If the slope is nonzero, $f$ on the sphere fails to be smooth at the poles. Therefore, for $f$ to be smooth on the sphere and satisfy $frac{partial^2 f}{partial theta^2} + frac{partial^2 f}{partial phi^2} = 0$, it has to be constant.






    share|cite|improve this answer









    $endgroup$













    • $begingroup$
      Ah, you beat me to an answer :)
      $endgroup$
      – Willie Wong
      Nov 1 '10 at 12:23










    • $begingroup$
      @Willie: But yours is more precisely expressed :)
      $endgroup$
      – Rahul
      Nov 1 '10 at 18:09



















    -1












    $begingroup$

    Partial differentiation in Laplacian does't works at all in all curved surfaces. SO the answer is No. However I can't say anything about $S^n$ in an continuous series






    share|cite|improve this answer









    $endgroup$













      Your Answer





      StackExchange.ifUsing("editor", function () {
      return StackExchange.using("mathjaxEditing", function () {
      StackExchange.MarkdownEditor.creationCallbacks.add(function (editor, postfix) {
      StackExchange.mathjaxEditing.prepareWmdForMathJax(editor, postfix, [["$", "$"], ["\\(","\\)"]]);
      });
      });
      }, "mathjax-editing");

      StackExchange.ready(function() {
      var channelOptions = {
      tags: "".split(" "),
      id: "69"
      };
      initTagRenderer("".split(" "), "".split(" "), channelOptions);

      StackExchange.using("externalEditor", function() {
      // Have to fire editor after snippets, if snippets enabled
      if (StackExchange.settings.snippets.snippetsEnabled) {
      StackExchange.using("snippets", function() {
      createEditor();
      });
      }
      else {
      createEditor();
      }
      });

      function createEditor() {
      StackExchange.prepareEditor({
      heartbeatType: 'answer',
      autoActivateHeartbeat: false,
      convertImagesToLinks: true,
      noModals: true,
      showLowRepImageUploadWarning: true,
      reputationToPostImages: 10,
      bindNavPrevention: true,
      postfix: "",
      imageUploader: {
      brandingHtml: "Powered by u003ca class="icon-imgur-white" href="https://imgur.com/"u003eu003c/au003e",
      contentPolicyHtml: "User contributions licensed under u003ca href="https://creativecommons.org/licenses/by-sa/3.0/"u003ecc by-sa 3.0 with attribution requiredu003c/au003e u003ca href="https://stackoverflow.com/legal/content-policy"u003e(content policy)u003c/au003e",
      allowUrls: true
      },
      noCode: true, onDemand: true,
      discardSelector: ".discard-answer"
      ,immediatelyShowMarkdownHelp:true
      });


      }
      });














      draft saved

      draft discarded


















      StackExchange.ready(
      function () {
      StackExchange.openid.initPostLogin('.new-post-login', 'https%3a%2f%2fmath.stackexchange.com%2fquestions%2f8510%2fharmonic-functions-on-s2%23new-answer', 'question_page');
      }
      );

      Post as a guest















      Required, but never shown

























      3 Answers
      3






      active

      oldest

      votes








      3 Answers
      3






      active

      oldest

      votes









      active

      oldest

      votes






      active

      oldest

      votes









      7












      $begingroup$

      Since $f$ is a smooth function from $S^2to mathbb{R}$, it is a smooth function on your coordinate chart. It is easy to see that by periodicity, you also have that $f$ is a smooth function from $S^1times(0,pi)tomathbb{R}$.



      Since $f$ is smooth on $S^2$, the limits



      $$ lim_{phi to 0, pi} frac{1}{sin phi} frac{partial}{partial theta} f $$



      need to exist and be finite: this is the requirement that $f$ is smooth in "the other chart". Therefore you have that $partial_theta f to 0$ as $phi$ approach either boundary.



      Your equation, however, requires that $partial_theta f$ be a harmonic function on $S^1times (0,pi)$, a compact domain with boundary. Using the maximum principle, you immediately have that $partial_theta f = 0$ everywhere. Plugging back into the equation, this implies $partial^2f/partial phi^2 = 0$.



      And so $f$ must be a linear function in $phi$. And we now apply the smoothness constraint yet again:



      At the north pole, smoothness will require that



      $$ lim_{phi to 0} partial_phi f(theta,phi) = lim_{phi to 0} frac{1}{sin phi} partial_theta f(theta + pi/2, phi) = 0$$



      which implies that the constant slope of the linear function $f(phi)$ is 0. And hence $f$ must be the constant function.






      share|cite|improve this answer









      $endgroup$


















        7












        $begingroup$

        Since $f$ is a smooth function from $S^2to mathbb{R}$, it is a smooth function on your coordinate chart. It is easy to see that by periodicity, you also have that $f$ is a smooth function from $S^1times(0,pi)tomathbb{R}$.



        Since $f$ is smooth on $S^2$, the limits



        $$ lim_{phi to 0, pi} frac{1}{sin phi} frac{partial}{partial theta} f $$



        need to exist and be finite: this is the requirement that $f$ is smooth in "the other chart". Therefore you have that $partial_theta f to 0$ as $phi$ approach either boundary.



        Your equation, however, requires that $partial_theta f$ be a harmonic function on $S^1times (0,pi)$, a compact domain with boundary. Using the maximum principle, you immediately have that $partial_theta f = 0$ everywhere. Plugging back into the equation, this implies $partial^2f/partial phi^2 = 0$.



        And so $f$ must be a linear function in $phi$. And we now apply the smoothness constraint yet again:



        At the north pole, smoothness will require that



        $$ lim_{phi to 0} partial_phi f(theta,phi) = lim_{phi to 0} frac{1}{sin phi} partial_theta f(theta + pi/2, phi) = 0$$



        which implies that the constant slope of the linear function $f(phi)$ is 0. And hence $f$ must be the constant function.






        share|cite|improve this answer









        $endgroup$
















          7












          7








          7





          $begingroup$

          Since $f$ is a smooth function from $S^2to mathbb{R}$, it is a smooth function on your coordinate chart. It is easy to see that by periodicity, you also have that $f$ is a smooth function from $S^1times(0,pi)tomathbb{R}$.



          Since $f$ is smooth on $S^2$, the limits



          $$ lim_{phi to 0, pi} frac{1}{sin phi} frac{partial}{partial theta} f $$



          need to exist and be finite: this is the requirement that $f$ is smooth in "the other chart". Therefore you have that $partial_theta f to 0$ as $phi$ approach either boundary.



          Your equation, however, requires that $partial_theta f$ be a harmonic function on $S^1times (0,pi)$, a compact domain with boundary. Using the maximum principle, you immediately have that $partial_theta f = 0$ everywhere. Plugging back into the equation, this implies $partial^2f/partial phi^2 = 0$.



          And so $f$ must be a linear function in $phi$. And we now apply the smoothness constraint yet again:



          At the north pole, smoothness will require that



          $$ lim_{phi to 0} partial_phi f(theta,phi) = lim_{phi to 0} frac{1}{sin phi} partial_theta f(theta + pi/2, phi) = 0$$



          which implies that the constant slope of the linear function $f(phi)$ is 0. And hence $f$ must be the constant function.






          share|cite|improve this answer









          $endgroup$



          Since $f$ is a smooth function from $S^2to mathbb{R}$, it is a smooth function on your coordinate chart. It is easy to see that by periodicity, you also have that $f$ is a smooth function from $S^1times(0,pi)tomathbb{R}$.



          Since $f$ is smooth on $S^2$, the limits



          $$ lim_{phi to 0, pi} frac{1}{sin phi} frac{partial}{partial theta} f $$



          need to exist and be finite: this is the requirement that $f$ is smooth in "the other chart". Therefore you have that $partial_theta f to 0$ as $phi$ approach either boundary.



          Your equation, however, requires that $partial_theta f$ be a harmonic function on $S^1times (0,pi)$, a compact domain with boundary. Using the maximum principle, you immediately have that $partial_theta f = 0$ everywhere. Plugging back into the equation, this implies $partial^2f/partial phi^2 = 0$.



          And so $f$ must be a linear function in $phi$. And we now apply the smoothness constraint yet again:



          At the north pole, smoothness will require that



          $$ lim_{phi to 0} partial_phi f(theta,phi) = lim_{phi to 0} frac{1}{sin phi} partial_theta f(theta + pi/2, phi) = 0$$



          which implies that the constant slope of the linear function $f(phi)$ is 0. And hence $f$ must be the constant function.







          share|cite|improve this answer












          share|cite|improve this answer



          share|cite|improve this answer










          answered Nov 1 '10 at 11:49









          Willie WongWillie Wong

          55.8k10111212




          55.8k10111212























              7












              $begingroup$

              If I understand your question correctly, you are aware that $frac{partial^2 f}{partial theta^2} + frac{partial^2 f}{partial phi^2}$ is not actually the Laplacian on a sphere, but you want to know whether there are any nontrivial smooth functions on $S^2$ which are harmonic with respect to this modified Laplacian. The answer is no.



              A function $f(theta,phi)$ which satisfies $frac{partial^2 f}{partial theta^2} + frac{partial^2 f}{partial phi^2} = 0$ is really a harmonic function on the rectangle $[0,2pi] times [0,pi]$, which is mapped onto the sphere via $g^{-1}$. Continuity on the sphere requires that $f(theta,0) = text{const}$ (continuity at the north pole), $f(theta,pi) = text{const}$ (continuity at the south pole), and $f(0,phi) = f(2pi,phi) = h(phi)$ for some $h:[0,pi]rightarrow mathbb{R}$ (continuity across the prime meridian). For any $h$, the boundary conditions are independent of $theta$, so $f$ is a function of $phi$ alone. But then $frac{partial^2 f}{partial phi^2} = 0$ implies that $f$ is linear in $phi$. If the slope is nonzero, $f$ on the sphere fails to be smooth at the poles. Therefore, for $f$ to be smooth on the sphere and satisfy $frac{partial^2 f}{partial theta^2} + frac{partial^2 f}{partial phi^2} = 0$, it has to be constant.






              share|cite|improve this answer









              $endgroup$













              • $begingroup$
                Ah, you beat me to an answer :)
                $endgroup$
                – Willie Wong
                Nov 1 '10 at 12:23










              • $begingroup$
                @Willie: But yours is more precisely expressed :)
                $endgroup$
                – Rahul
                Nov 1 '10 at 18:09
















              7












              $begingroup$

              If I understand your question correctly, you are aware that $frac{partial^2 f}{partial theta^2} + frac{partial^2 f}{partial phi^2}$ is not actually the Laplacian on a sphere, but you want to know whether there are any nontrivial smooth functions on $S^2$ which are harmonic with respect to this modified Laplacian. The answer is no.



              A function $f(theta,phi)$ which satisfies $frac{partial^2 f}{partial theta^2} + frac{partial^2 f}{partial phi^2} = 0$ is really a harmonic function on the rectangle $[0,2pi] times [0,pi]$, which is mapped onto the sphere via $g^{-1}$. Continuity on the sphere requires that $f(theta,0) = text{const}$ (continuity at the north pole), $f(theta,pi) = text{const}$ (continuity at the south pole), and $f(0,phi) = f(2pi,phi) = h(phi)$ for some $h:[0,pi]rightarrow mathbb{R}$ (continuity across the prime meridian). For any $h$, the boundary conditions are independent of $theta$, so $f$ is a function of $phi$ alone. But then $frac{partial^2 f}{partial phi^2} = 0$ implies that $f$ is linear in $phi$. If the slope is nonzero, $f$ on the sphere fails to be smooth at the poles. Therefore, for $f$ to be smooth on the sphere and satisfy $frac{partial^2 f}{partial theta^2} + frac{partial^2 f}{partial phi^2} = 0$, it has to be constant.






              share|cite|improve this answer









              $endgroup$













              • $begingroup$
                Ah, you beat me to an answer :)
                $endgroup$
                – Willie Wong
                Nov 1 '10 at 12:23










              • $begingroup$
                @Willie: But yours is more precisely expressed :)
                $endgroup$
                – Rahul
                Nov 1 '10 at 18:09














              7












              7








              7





              $begingroup$

              If I understand your question correctly, you are aware that $frac{partial^2 f}{partial theta^2} + frac{partial^2 f}{partial phi^2}$ is not actually the Laplacian on a sphere, but you want to know whether there are any nontrivial smooth functions on $S^2$ which are harmonic with respect to this modified Laplacian. The answer is no.



              A function $f(theta,phi)$ which satisfies $frac{partial^2 f}{partial theta^2} + frac{partial^2 f}{partial phi^2} = 0$ is really a harmonic function on the rectangle $[0,2pi] times [0,pi]$, which is mapped onto the sphere via $g^{-1}$. Continuity on the sphere requires that $f(theta,0) = text{const}$ (continuity at the north pole), $f(theta,pi) = text{const}$ (continuity at the south pole), and $f(0,phi) = f(2pi,phi) = h(phi)$ for some $h:[0,pi]rightarrow mathbb{R}$ (continuity across the prime meridian). For any $h$, the boundary conditions are independent of $theta$, so $f$ is a function of $phi$ alone. But then $frac{partial^2 f}{partial phi^2} = 0$ implies that $f$ is linear in $phi$. If the slope is nonzero, $f$ on the sphere fails to be smooth at the poles. Therefore, for $f$ to be smooth on the sphere and satisfy $frac{partial^2 f}{partial theta^2} + frac{partial^2 f}{partial phi^2} = 0$, it has to be constant.






              share|cite|improve this answer









              $endgroup$



              If I understand your question correctly, you are aware that $frac{partial^2 f}{partial theta^2} + frac{partial^2 f}{partial phi^2}$ is not actually the Laplacian on a sphere, but you want to know whether there are any nontrivial smooth functions on $S^2$ which are harmonic with respect to this modified Laplacian. The answer is no.



              A function $f(theta,phi)$ which satisfies $frac{partial^2 f}{partial theta^2} + frac{partial^2 f}{partial phi^2} = 0$ is really a harmonic function on the rectangle $[0,2pi] times [0,pi]$, which is mapped onto the sphere via $g^{-1}$. Continuity on the sphere requires that $f(theta,0) = text{const}$ (continuity at the north pole), $f(theta,pi) = text{const}$ (continuity at the south pole), and $f(0,phi) = f(2pi,phi) = h(phi)$ for some $h:[0,pi]rightarrow mathbb{R}$ (continuity across the prime meridian). For any $h$, the boundary conditions are independent of $theta$, so $f$ is a function of $phi$ alone. But then $frac{partial^2 f}{partial phi^2} = 0$ implies that $f$ is linear in $phi$. If the slope is nonzero, $f$ on the sphere fails to be smooth at the poles. Therefore, for $f$ to be smooth on the sphere and satisfy $frac{partial^2 f}{partial theta^2} + frac{partial^2 f}{partial phi^2} = 0$, it has to be constant.







              share|cite|improve this answer












              share|cite|improve this answer



              share|cite|improve this answer










              answered Nov 1 '10 at 11:46









              RahulRahul

              33.2k568173




              33.2k568173












              • $begingroup$
                Ah, you beat me to an answer :)
                $endgroup$
                – Willie Wong
                Nov 1 '10 at 12:23










              • $begingroup$
                @Willie: But yours is more precisely expressed :)
                $endgroup$
                – Rahul
                Nov 1 '10 at 18:09


















              • $begingroup$
                Ah, you beat me to an answer :)
                $endgroup$
                – Willie Wong
                Nov 1 '10 at 12:23










              • $begingroup$
                @Willie: But yours is more precisely expressed :)
                $endgroup$
                – Rahul
                Nov 1 '10 at 18:09
















              $begingroup$
              Ah, you beat me to an answer :)
              $endgroup$
              – Willie Wong
              Nov 1 '10 at 12:23




              $begingroup$
              Ah, you beat me to an answer :)
              $endgroup$
              – Willie Wong
              Nov 1 '10 at 12:23












              $begingroup$
              @Willie: But yours is more precisely expressed :)
              $endgroup$
              – Rahul
              Nov 1 '10 at 18:09




              $begingroup$
              @Willie: But yours is more precisely expressed :)
              $endgroup$
              – Rahul
              Nov 1 '10 at 18:09











              -1












              $begingroup$

              Partial differentiation in Laplacian does't works at all in all curved surfaces. SO the answer is No. However I can't say anything about $S^n$ in an continuous series






              share|cite|improve this answer









              $endgroup$


















                -1












                $begingroup$

                Partial differentiation in Laplacian does't works at all in all curved surfaces. SO the answer is No. However I can't say anything about $S^n$ in an continuous series






                share|cite|improve this answer









                $endgroup$
















                  -1












                  -1








                  -1





                  $begingroup$

                  Partial differentiation in Laplacian does't works at all in all curved surfaces. SO the answer is No. However I can't say anything about $S^n$ in an continuous series






                  share|cite|improve this answer









                  $endgroup$



                  Partial differentiation in Laplacian does't works at all in all curved surfaces. SO the answer is No. However I can't say anything about $S^n$ in an continuous series







                  share|cite|improve this answer












                  share|cite|improve this answer



                  share|cite|improve this answer










                  answered Dec 23 '18 at 14:50







                  user628840





































                      draft saved

                      draft discarded




















































                      Thanks for contributing an answer to Mathematics Stack Exchange!


                      • Please be sure to answer the question. Provide details and share your research!

                      But avoid



                      • Asking for help, clarification, or responding to other answers.

                      • Making statements based on opinion; back them up with references or personal experience.


                      Use MathJax to format equations. MathJax reference.


                      To learn more, see our tips on writing great answers.




                      draft saved


                      draft discarded














                      StackExchange.ready(
                      function () {
                      StackExchange.openid.initPostLogin('.new-post-login', 'https%3a%2f%2fmath.stackexchange.com%2fquestions%2f8510%2fharmonic-functions-on-s2%23new-answer', 'question_page');
                      }
                      );

                      Post as a guest















                      Required, but never shown





















































                      Required, but never shown














                      Required, but never shown












                      Required, but never shown







                      Required, but never shown

































                      Required, but never shown














                      Required, but never shown












                      Required, but never shown







                      Required, but never shown







                      Popular posts from this blog

                      How do I know what Microsoft account the skydrive app is syncing to?

                      Grease: Live!

                      When does type information flow backwards in C++?